This general inequality maybe is true? $\sum_{i=1}^{n}\frac{i}{1+a_{1}+\cdots+a_{i}}<\frac{n}{2}\sqrt{\sum_{i=1}^{n}\frac{1}{a_{i}}}$

Let $a_{1},a_{2},\ldots,a_{n}>0$ and prove or disprove $$\dfrac{1}{1+a_{1}}+\dfrac{2}{1+a_{1}+a_{2}}+\cdots+\dfrac{n}{1+a_{1}+a_{2}+\cdots+a_{n}}\le\dfrac{n}{2}\sqrt{\dfrac{1}{a_{1}}+\dfrac{1}{a_{2}}+\cdots+\dfrac{1}{a_{n}}}\tag{1}$$

This problem from when I solve this two variable inequality

since $n=1$ it is clear $$\dfrac{1}{1+a_{1}}\le\dfrac{1}{2}\sqrt{\dfrac{1}{a_{1}}}$$ because $1+a_{1}\ge 2\sqrt{a_{1}}$

$n=2$ case,can see this links my answer.

For general

simaler this two variable inequality methods, then I use Cauchy-Schwarz inequality we have $$\left(\sum_{i=1}^{n}\dfrac{1}{1+a_{1}+\cdots+a_{i}}\right)^2\le\left(\sum_{i=1}^{n}\dfrac{1}{a_{i}}\right)\cdot\left(\sum_{i=1}^{n}\dfrac{i^2a_{i}}{(1+a_{1}+a_{2}+\cdots+a_{i})^2}\right)$$

it suffices to show that $$\sum_{i=1}^{n}\dfrac{i^2a_{i}}{(1+a_{1}+\cdots+a_{i})^2}\le\dfrac{n^2}{4}\tag{2}$$ it seem hard.

because I tried following also fail; $$\sum_{i=1}^{n}\dfrac{i^2a_{i}}{(1+a_{1}+\cdots+a_{i})^2}<\sum_{i=1}^{n}i^2\left(\dfrac{1}{1+a_{1}+\cdots+a_{i-1}}-\dfrac{1}{1+a_{1}+a_{2}+\cdots+a_{i}}\right)$$ and use Abel transformation.not can to prove $(2)$,

Note $(1)$ Left side hand was simaler Hardy's inequality when $p=-1$,But there are different problem.

EDIT:Numerical tests $(2)$ is not right.so my idea can't works


suppose $n=k, \\ \dfrac{1}{1+a_{1}}+\dfrac{2}{1+a_{1}+a_{2}}+\cdots+\dfrac{k}{1+a_{1}+a_{2}+\cdots+a_{k}} \\ \le\dfrac{k}{2}\sqrt{\dfrac{1}{a_{1}}+\dfrac{1}{a_{2}}+\cdots+\dfrac{1}{a_{k}}}$

when $n=k+1 $

LHS=$\dfrac{1}{1+a_{1}}+\dfrac{2}{1+a_{1}+a_{2}}+\cdots+\dfrac{k}{1+a_{1}+a_{2}+\cdots+a_{k}}+\dfrac{k+1}{1+a_{1}+a_{2}+\cdots+a_{k+1}} \\<\dfrac{k}{2}\sqrt{\dfrac{1}{a_{1}}+\dfrac{1}{a_{2}}+\cdots+\dfrac{1}{a_{k}}}+\dfrac{k+1}{1+a_{1}+a_{2}+\cdots+a_{k+1}} \\ <\dfrac{k}{2}\sqrt{\dfrac{1}{a_{1}}+\dfrac{1}{a_{2}}+\cdots+\dfrac{1}{a_{k}}+\dfrac{1}{a_{k+1}}}+\dfrac{k+1}{1+a_{1}+a_{2}+\cdots+a_{k+1}}$

RHS$=\dfrac{k}{2}\sqrt{\dfrac{1}{a_{1}}+\dfrac{1}{a_{2}}+\cdots+\dfrac{1}{a_{k}}+\dfrac{1}{a_{k+1}}}+\dfrac{1}{2}\sqrt{\dfrac{1}{a_{1}}+\dfrac{1}{a_{2}}+\cdots+\dfrac{1}{a_{k}}+\dfrac{1}{a_{k+1}}}$

so is remains: $ \dfrac{k+1}{1+a_{1}+a_{2}+\cdots+a_{k+1}} \\ \le \dfrac{1}{2}\sqrt{\dfrac{1}{a_{1}}+\dfrac{1}{a_{2}}+\cdots+\dfrac{1}{a_{k}}+\dfrac{1}{a_{k+1}}}$

$1+a_{1}+a_{2}+\cdots+a_{k+1} \ge 2\sqrt{a_{1}+a_{2}+\cdots+a_{k+1}} \\ \implies \dfrac{k+1}{1+a_{1}+a_{2}+\cdots+a_{k+1}} \le \dfrac{k+1}{2\sqrt{a_{1}+a_{2}+\cdots+a_{k+1}}} \le \dfrac{1}{2}\sqrt{\dfrac{1}{a_{1}}+\dfrac{1}{a_{2}}+\cdots+\dfrac{1}{a_{k}}+\dfrac{1}{a_{k+1}}} \\ \iff \dfrac{(k+1)^2}{\sum_{i=1}^{k+1} a_i} \le \sum_{i=1}^{k+1} \dfrac{1}{a_i}$

the last one is true ,$HM\le AM$

QED